The easiest logic game question type to answer on the LSAT is the possible listing/assignment question. It’s usually the first question of the set. You don’t need to refer to the game board to answer it; you simply apply each condition to the answer choices and eliminate the four answers that violate a condition.

These questions ask you for the answer that provides an accurate ordering or grouping of the game pieces. The five answers supply five potential orders or groups for the logic games. Four of the orders or groups violate at least one condition and are therefore not possible. Only one provides a possibility that doesn’t violate a rule.

The key to answering every possible listing/assignment question correctly is to read a rule and then eliminate the answer (or answers) that violates it rather than reading an answer choice and finding the rule(s) it violates. Here’s a sample possible listing/assignment question for the disc jockey scenario to demonstrate what this means.

Which of the following could be the programming order of the disc jockeys from first to last?

  • (A)Penelope, Marco, Lucy, Jamal, Gary, Harriet

  • (B)Lucy, Penelope, Marco, Harriet, Jamal, Gary

  • (C)Harriet, Lucy, Penelope, Jamal, Gary, Marco

  • (D)Gary, Marco, Penelope, Lucy, Harriet, Jamal

  • (E)Gary, Lucy, Jamal, Penelope, Marco, Harriet

This is an ordering problem, so the question asks for a possible order. When you see this question type for a grouping problem, it will likely ask for a possible assignment or matching of elements to groups. Here are the steps to answering this question:

  1. Read the first condition.

    It states that both L and M are before H. Look through the five answer choices to find the one that doesn’t place L and M before H. Choice (C) puts H first, so neither L nor M is before H. Cross out Choice (C).

  2. Read the second condition.

    Both L and P are before J. Choice (E) positions J between L and P, so it can’t be a possible order. Eliminate Choice (E).

  3. The third condition pertains to when M is before P.

    Of the remaining answers, only Choice (D) positions M before P. H is definitely after G in Choice (D), so that answer violates the third rule and is incorrect.

  4. Check the last condition.

    It’s an easy one to evaluate. Gary can’t be last, but Choice (B) puts Gary at the end. So Choice (B) must be wrong.

  5. Do a quick rule check for the remaining answer Choice (A).

    L and M are before H, L and P are before J, the third condition doesn’t apply, and Gary isn’t last. Its order is entirely possible.

Choice (A) is the answer.

See how easy this question type is when you approach it methodically? The good news is that virtually every logic game begins with one of these easy question types, so you’re bound to answer the first question of every problem set correctly.

Note that you don’t need to use the game board to answer this question. So even if you’re having trouble setting up a board for a problem set, at least attempt to answer the first question.

About This Article

This article is from the book:

About the book authors:

Lisa Zimmer Hatch served as VP of The Center for Legal Studies, where she created standardized test preparation. Currently, she is an Independent College Counselor and president of College Primers.

Scott A. Hatch develops courses for a variety of careers and assists those seeking advanced degrees in law, business, and other professions.

Lisa Zimmer Hatch served as VP of The Center for Legal Studies, where she created standardized test preparation. Currently, she is an Independent College Counselor and president of College Primers.

Scott A. Hatch develops courses for a variety of careers and assists those seeking advanced degrees in law, business, and other professions.

This article can be found in the category: